Đến nội dung

Ego nội dung

Có 288 mục bởi Ego (Tìm giới hạn từ 07-06-2020)



Sắp theo                Sắp xếp  

#616154 $$\frac{1}{2ne}<\frac{1...

Đã gửi bởi Ego on 20-02-2016 - 22:52 trong Bất đẳng thức và cực trị

Với $n = 1$, bđt đúng. Xét $n > 1$
Bổ đề 1. $\lim_{n\to+\infty}{(1 - \frac{1}{n})^{n}} = \frac{1}{e}$
Chứng minh. Một giới hạn quen thuộc là $\lim_{k\to+\infty}{(1 + \frac{1}{k})^{k + 1}} = e$. Thay $k$ bởi $n - 1$, có
$\lim_{n\to+\infty}(\frac{n}{n - 1})^{n} = e \implies \lim_{n\to+\infty}(1 - \frac{1}{n})^{n} = \frac{1}{e}$


 

BĐT cần chứng minh tương đương với $\frac{2n - 1}{2n} > e(1 - \frac{1}{n})^{n} > \frac{n - 1}{n}$
Lấy $\ln$, có $\ln (2n - 1) - \ln (2n) > 1 + n\ln (1 - \frac{1}{n}) > \ln (n - 1) - \ln (n)$
1) $\ln (2n - 1) - \ln (2n) > 1 + n\ln (1 - \frac{1}{n}) \iff \ln(2n - 1) - \ln(2n) - 1 - n\ln(1 - \frac{1}{n}) > 0$. Xét hàm số $f(n) = \ln(2n - 1) - \ln(2n) - 1 - n\ln(1 - \frac{1}{n}) = \ln(2n - 1) - \ln(2n) - 1 - n\ln(n - 1) + n\ln(n)$ với $n \in \mathbb{N}, n \neq 1$
$f'(n) = \frac{2}{2n - 1} - \frac{1}{n} - \ln(n - 1) - \frac{n}{n - 1} + \ln(n) + 1 = \frac{1}{n(2n - 1)} - \frac{1}{n - 1} + \ln(n) - \ln(n - 1)$
$f''(n) = \frac{1- 4n}{(2n^{2} - n)^{2}} + \frac{1}{(n - 1)^{2}} + \frac{1}{n} - \frac{1}{n - 1} = \frac{5n^{2} - 5n + 1}{(2n - 1)^{2}n^{2}(n - 1)^{2}} > 0 \forall n \ge 1$
Do đó $f''(n)$ là hàm tăng và liên tục trên $[2; +\infty)$ nên $f'(n) < \lim_{n\to+\infty}f'(n) = 0$. Do đó hàm $f(n)$ là hàm giảm trên $[2; +\infty)$
$\implies f(n) > \lim_{n\to+\infty}f(n) = 0$ (từ bổ đề 1)

2) $1 + n\ln(n - 1) - n\ln(n) - \ln(n - 1) + \ln(n) > 0$. Xét $g(n) = 1 + n\ln(n - 1) - n\ln(n) - \ln(n - 1) + \ln(n)$ trên $[2; +\infty)$
Có $g'(n) = \ln(n - 1) - \ln(n) + \frac{1}{n}$ và $g''(n) = \frac{1}{n - 1} - \frac{1}{n} - \frac{1}{n^{2}} = \frac{1}{n^{2}(n - 1)} > 0$.
Từ đó có $g'(n)$ đồng biến trên $[2; +\infty)$ hay $g'(n) < \lim_{n\to+\infty}g'(n) = 0$. Do đó $g(n)$ nghịch biến trên $[2; +\infty)$
$g(n) > \lim_{n\to+\infty}g(n) = 0$.
Vậy bất đẳng thức luôn đúng.
Lời giải xấu xí quá -.-



#623928 $$\frac{a^{2}b}{c} + \frac...

Đã gửi bởi Ego on 31-03-2016 - 21:55 trong Bất đẳng thức và cực trị

Lời giải 1 dòng.
$$\frac{a^{2}b}{c} + \frac{b^{2}c}{a} + \frac{c^{2}a}{b} - (a^{2} + b^{2} + c^{2}) = \frac{[a(b - c)]^{2}}{bc} + \frac{[c(a - b)]^{2}}{ab} + \frac{(a - b)(b - c)[a^{2} + b(a - c)]}{ab} \ge 0$$




#623720 $$\frac{a^{2}b}{c} + \frac...

Đã gửi bởi Ego on 30-03-2016 - 21:27 trong Bất đẳng thức và cực trị

Lâu rồi không làm BĐT, càng ngày càng dở :'(. Tặng các bạn.
Cho $a \ge b \ge c > 0$. Chứng minh rằng:
$$\frac{a^{2}b}{c} + \frac{b^{2}c}{a} + \frac{c^{2}a}{b} \ge a^{2} + b^{2} + c^{2}$$




#625884 $$\lim_{n\to \infty}S(2^{n}) = +...

Đã gửi bởi Ego on 08-04-2016 - 16:25 trong Số học

Ký hiệu $S(n)$ là tổng các chữ số của $n$ trong hệ thập phân. Chứng minh rằng
$$\lim_{n\to \infty}S(2^{n}) = +\infty$$

Nguồn: Facebook




#627254 $$\sqrt{a^{2} + a + 1} + \sqrt{a...

Đã gửi bởi Ego on 15-04-2016 - 17:04 trong Bất đẳng thức và cực trị

Cho $a$ là một số thực. Chứng minh rằng:
$$\sqrt{a^{2} + a + 1} + \sqrt{a^{2} - a + 1} \ge 2$$




#627319 $$\sqrt{a^{2} + a + 1} + \sqrt{a...

Đã gửi bởi Ego on 15-04-2016 - 21:16 trong Bất đẳng thức và cực trị

(y) Ngoài ra còn có thể dùng bất đẳng thức Cauchy - Schwazrt mở rộng như sau $\sqrt{a^{2} + b^{2}} + \sqrt{c^{2} + d^{2}} \ge \sqrt{(a + c)^{2} + (b + d)^{2}}$ đúng với mọi SỐ THỰC $a, b, c, d$:
Áp dụng, ta có $\sqrt{(a + \frac{1}{2})^{2} + \frac{3}{4}} + \sqrt{(\frac{1}{2} - a)^{2} + \frac{3}{4}} \ge \sqrt{(a + \frac{1}{2} + \frac{1}{2} - a)^{2} + 3} = 2$



#624576 $$\sum \frac{a^{2}b}{c} +...

Đã gửi bởi Ego on 03-04-2016 - 19:59 trong Bất đẳng thức và cực trị

Cho $a \ge b \ge c \ge d > 0$. Chứng minh rằng
$$\frac{a^{2}b}{c} + \frac{b^{2}c}{d} + \frac{c^{2}d}{a} + \frac{d^{2}a}{b} \ge a^{2} + b^{2} + c^{2} + d^{2}$$




#617249 $\gcd (n,1)+ \gcd (n,2)+ \cdots + \gcd (n,n) = 3n-3....

Đã gửi bởi Ego on 27-02-2016 - 20:21 trong Số học

Gọi $d=(k,n)$ thì $(\frac{k}{d},\frac{n}{d})=1$ nên ta có thể viết lại vế trái dưới dạng $\sum_{d|n}d\phi (\frac{n}{d})$

Ta áp dụng công thức siêu kinh điển : Với $f,g$ là hai hàm nhân tính thì $\sum_{d|n}f(d)g(\frac{n}{d})= \prod _{p|n}(\sum_{k=0}^{v_p(n)}f(p^k)g(p^{v_p(n)-k}))$

Ở đây ta chọn $f(x)=x,g(x)=\phi (x)$ thì $\sum_{d|n}d\phi (\frac{n}{d})= \prod _{p|n}(\sum_{k=0}^{v_p(n)}p^k\phi (p^{v_p(n)-k}))= \prod _{p|n}(\sum_{k=0}^{v_p(n)}p^k.p^{v_p(n)-k-1}(p-1))$

$= \prod _{p|n}(\sum_{k=0}^{v_p(n)}p^{v_p(n)-1}(p-1))= \prod _{p|n}((v_p(n)+1)p^{v_p(n)-1}(p-1))=3n-3$                                            $(1)$

Do đó, nếu tồn tại $p|n$ mà $v_p(n)\geq 2$ thì $p^{v_p(n)-1}|3\Rightarrow p=3$ và  $v_3(n)= 2$ , khi đó $(v_3(n)+1)3^{v_3(n)-1}(3-1)=3.3.2$ chia hết cho $9$ nên vô lí

Suy ra $v_p(n)=1$ với mọi $p|n$

Giả sử $n=p_1p_2...p_k$ thì từ  $(1)$ suy ra $2^k(p_1-1)(p_2-1)...(p_k-1)=3p_1p_2...p_k-3$.

Đánh giá bđt đơn giản ta được $k=2$ , dẫn đến $4(p_1-1)(p_2-1)=3p_1p_2-3$

Phương trình cơ bản , kết quả là $p_1=5$, $p_2=13$ và $n=65$

 

p/s: $jinbe$ cho xin cái lời giải cái ._. làm thế này trâu quá ._.

Hình như không đúng. Mình chứng minh được số cần tìm là chẵn và square-free, bị đứng một chỗ. Và bạn thử lại xem. $n = 2p$ luôn thỏa mãn với $p$ là số nguyên tố lẻ :-) ($n = 10, n = 14$ vẫn thỏa mãn)




#617290 $\gcd (n,1)+ \gcd (n,2)+ \cdots + \gcd (n,n) = 3n-3....

Đã gửi bởi Ego on 27-02-2016 - 23:24 trong Số học

To @Visitor: Mình lần đầu đọc qua tính chất này của hàm nhân tính @@. Bạn giới thiệu cho mình vài tài liệu được không?
:-P Cách của mình thủ công hơn nhiều, biện luận và có được $n = 2p_{1}^{\alpha}p_{2}\cdots$ và biện luận trường hợp cho $\alpha = 1$ khá dài dòng  :(




#615650 $\left \lfloor nx \right \rfloor\geq \sum_...

Đã gửi bởi Ego on 17-02-2016 - 22:00 trong Số học

Bài này cũng khá dễ. Để ý bất đẳng thức $ix \ge \left \lfloor ix \right \rfloor$ (cái này từ định nghĩa, để ý đẳng thức xảy ra khi $x$ nguyên)
Do đó $\frac{\left \lceil ix \right \rceil}{i} \le x$. Cho $i$ chạy từ $1$ đến $n$ rồi cộng lại, ta có bđt cần cm.
Đẳng thức xảy ra khi và chỉ khi $x$ nguyên.




#620412 $\left\{\begin{matrix} 7\nmid ab(a+b)...

Đã gửi bởi Ego on 15-03-2016 - 20:00 trong Số học

Theo đề bài, $\text{gcd}(a, 7) = \text{gcd}(b, 7) = 1$. Kí hiệu $t = b^{-1} \pmod{7^{3}} \implies bt \equiv 1\pmod{7^{3}} \implies bt \equiv 1\pmod{7} \implies t = b^{-1} \pmod{7}$
Có $7\nmid t^{3}.a^{2}b + t^{3}ab^{2}$ hay $7\nmid (at)^{2} + at$.
Và $7^{3}\mid (at)^{2} + at + 1$. Đặt $u = at$. 
Vậy ta có$7^{3}\mid u^{2} + u + 1 \implies 7^{3} \mid (2u + 1)^{2} + 3$ (do $\text{gcd}(4, 7) = 1$)
Giải ra ta sẽ có $2u + 1 \equiv \pm 37\pmod{7^{3}}$. Hay $u \equiv 18\pmod{7^{3}}$ hoặc $u \equiv -19 \pmod{7^{3}}$
Thế vào điều kiện còn lại ta thấy đều thỏa hai điều kiện còn lại.
Từ đó suy ra $ab^{-1} \equiv 18; -19 \pmod{7^{3}} \implies a \equiv 18b; -19b\pmod{7^{3}}$.




#623410 $\left\{\begin{matrix} ab+1\vdots c...

Đã gửi bởi Ego on 29-03-2016 - 17:12 trong Số học

Do $ab + bc + ca + 1 \vdots abc$ nên $ab + bc + ca + 1 = h.abc$. Anh chỉ xét $h = 1$ và $h \ge 2$ thôi, nhiêu đó đã đủ hết khả năng của $h$ rồi.




#622992 $\left\{\begin{matrix} ab+1\vdots c...

Đã gửi bởi Ego on 27-03-2016 - 18:19 trong Số học

WLOG, giả sử $a \ge b \ge c$
Từ phương trình ban đầu ta suy ra $ab + bc + ca + 1 \vdots abc$. Mặt khác, $a, b, c$ nguyên dương nên $ab + bc + ca + 1 \ge abc$.
Lại có $(a - 1)(b - 1)(c - 1) \ge 1 \iff abc + a + b + c \ge 2 + ab + bc + ca$ (*)
TH1. Nếu $ab + bc + ca + 1 \ge 2abc$ thì từ (*) ta có $a + b + c \ge 1 + abc = 1 + a(bc + 1 - 1) \ge 1 + a(b + c) > a + b + ac$ (từ BĐT $(b - 1)(c - 1) \ge 1$). Vô lí.
TH2. $ab + bc + ca + 1 = abc \iff 1 = \frac{1}{a} + \frac{1}{b} + \frac{1}{c} + \frac{1}{abc}$.
Đến đây kiểm tra thôi, nếu $c \ge 4$ thì $\frac{1}{a} + \frac{1}{b} + \frac{1}{c} + \frac{1}{abc} \le \frac{3}{4} + \frac{1}{64} < 1$

Do đó $c \le 3$.

  • Nếu $c = 2$ thì ta viết lại, $\frac{1}{a} + \frac{1}{b} + \frac{1}{2ab} = \frac{1}{2}$
    Nếu $b \ge 5$ tương tự ta có điều vô lí. Kiểm tra được $(b = 3 \implies a = 7)$; và $b = 4$ thì không có nghiệm.
  • Nếu $c = 3$ thì ta viết lại, $\frac{1}{a} + \frac{1}{b} + \frac{1}{3ab} = \frac{2}{3}$
    Nếu $b \ge 4$ thì tương tự có điều vô lí. Kiểm tra được PT này vô nghiệm.

Vậy ta có $(a, b, c) = (7, 3, 2)$ và các hoán vị là nghiệm.




#634725 $\lim_{n \to \propto } \frac{n}...

Đã gửi bởi Ego on 22-05-2016 - 16:43 trong Dãy số - Giới hạn

Lời giải bằng L'Hospital

 




#634511 $(n-1)!$ không chia hết cho $n^{2}$

Đã gửi bởi Ego on 21-05-2016 - 18:22 trong Số học

 

 

Vậy các số thỏa mãn đề bài là các số không thuộc tập hợp (8,9,p,2p)(8,9,p,2p) với pp là một số nguyên tố bất kỳ.

Anh nhớ là anh ghi vậy mà.
@Nguyen Trang Mai: Thì ngược lại thôi em




#634338 $(n-1)!$ không chia hết cho $n^{2}$

Đã gửi bởi Ego on 20-05-2016 - 20:24 trong Số học

Đã giải tại đây.




#626769 $(x + 1)(x + 2)\cdots (x + 2014) = (y + 1)(y + 2)\cdots (y + 4...

Đã gửi bởi Ego on 12-04-2016 - 16:18 trong Số học

Chứng minh rằng không có hai bộ $(x, y)$ nguyên dương nào thỏa mãn: $(x + 1)(x + 2)\cdots (x + 2014) = (y + 1)(y + 2)\cdots (y + 4028)$

Nguồn: China TST 3 Day 1 2014 Q3




#621181 $(x^2+1)^y-(x^2-1)^y=(2x)^y$

Đã gửi bởi Ego on 19-03-2016 - 17:38 trong Số học

Một lời giải xấu xí.
Viết lại PT, $(x^{2} + 1)^{y} = (x^{2} - 1)^{y} + (2x)^{y}$
Nếu $y \ge 3$, theo định lý Fermat lớn thì không tồn tại bộ nguyên dương. Điều đó nghĩa là một trong các số $x^{2} + 1; x^{2} - 1; 2x$ bằng $0$
Dễ thấy $x^{2} + 1$ và $2x$ khác $0$ nên chỉ có thể $x^{2} = 1$ hay $x = 1$. Thế lại ta có nó đúng với mọi $y \ge 3$
Nếu $y = 1$ thì suy ra $x = 1$
Nếu $y = 2$ thì viết lại pt $x^{4} + 2x^{2} + 1 = x^{4} - 2x^{2} + 1 + 4x^{2}$ điều này luôn đúng. Nghĩa là $(x, y) = (t, 2)$ là nghiệm với $t$ nguyên dương bất kì
Vậy ta có các nghiệm $(x, y) = (1, t), (t, 2)$.
P.s: mình nghĩ mình còn 1 lgiai nữa không dùng đến Fermat lớn nhưng hiện tại không ở nhà nên sẽ post sau :-D



#633610 $(z + 1)f(x + y) = f(xf(z) + y) + f(yf(z) + x)$

Đã gửi bởi Ego on 17-05-2016 - 06:26 trong Phương trình hàm

Tìm tất cả các hàm $f: \mathbb{R} \to \mathbb{R}$ sao cho $$(z + 1)f(x + y) = f(xf(z) + y) + f(yf(z) + x)$$
 




#633636 $(z + 1)f(x + y) = f(xf(z) + y) + f(yf(z) + x)$

Đã gửi bởi Ego on 17-05-2016 - 13:14 trong Phương trình hàm

Ừ, bài này khác APMO một chút :3 Ban đầu mình làm thì chép là $\mathbb{R}$, sau đó làm ra mới biết là làm một bài toán mới luôn (y) khá hay nên chia sẻ các bạn




#627235 ... q | $a^p - 1$ và q | $a^n - 1$

Đã gửi bởi Ego on 15-04-2016 - 12:55 trong Số học

Bài toán không đúng với $a = 2$, mình nghĩ $(a, p, n) = (2, 3, 5)$ là một ví dụ. Nếu đề bài sửa lại thành $a \ge 3$ thì ta chỉ việc chọn $q$ là ước nguyên tố của $a - 1$. Từ đó có $q\mid a - 1 \mid a^{p} - 1$ và $q\mid a - 1 \mid a^{n} - 1$.




#610905 [Số học] THPT tháng 11: Tìm $a,b,c$ thoả $a^2+b \mid b^2+...

Đã gửi bởi Ego on 25-01-2016 - 11:59 trong Thảo luận đề thi VMEO IV

Bộ $a,b,c$ đôi một nguyên tố cùng nhau nên bộ nghiệm thứ hai phải loại chứ bạn

Ừ nhỉ :3 Tớ quên không kiểm lại điều kiện đó. Thanks.




#610776 [Số học] THPT tháng 11: Tìm $a,b,c$ thoả $a^2+b \mid b^2+...

Đã gửi bởi Ego on 24-01-2016 - 18:58 trong Thảo luận đề thi VMEO IV

Sao mình nhớ trong đề thi là 'tìm tất cả các ước số nguyên tố của $a^2 + b$ mà không đồng dư modulo $7$' nhỉ? Mà không sao cả, đợt đó mình chú tâm bài bđt với cả cũng bận ôn thi nên quên cả hạn gửi bài  :P. Đây là lời giải của mình cho bài toán trên.
$$\begin{cases} (a^{2} + b)\mid (c^{2} + a) \\ (a^{2} + b)\mid (b^{4} - c^{2}) \end{cases} \implies (a^{2} + b)\mid (b^{4} + a) \implies (a^{2} + b)\mid (b^{8} - a^{2}) \implies (a^{2} + b)\mid ((b^{8} - a^{16}) + (a^{16} - a^{2}) \implies (a^{2} + b)\mid a^{2}(a^{14} - 1)$$
Dễ thấy $\text{gcd}(a^{2} + b; a^{2})\mid \text{gcd}(b; a^{2})\mid \text{gcd}(a^{2}; b^{2})\mid (\text{gcd}(a; b))^{2} = 1 \implies \text{gcd}(a^{2} + b; a^{2}) = 1$
$$\implies (a^{2} + b)\mid (a^{14} - 1)$$. Gọi $p$ là một ước nguyên tố bất kì của $a^2 + b$:
TH1: $p = 7$. Dễ thấy $\text{gcd}(p; a) = 1$, do đó theo định lý Fermat bé: $1 \equiv a^{14} \equiv a^{2} \pmod{7}$. Từ đó có $7\mid (a^{2} - 1)$. Theo bổ đề LTE thì $v_{7}(a^{14} - 1) = v_{7}(a^{2} - 1) + 1$. Do đó $v_{7}(a^{2} + b) \le v_{7}(7(a^{2} - 1))$
TH2: $p \neq 7$. Nếu $p\mid \frac{a^{14} - 1}{a^{2} - 1}$, thì theo một bổ đề cũ: "Cho $L$ là một số nguyên dương và $p$ là một số nguyên tố sao cho $L\mid \frac{x^{p} - 1}{x - 1}$ thì $L \equiv 0; 1 \pmod{p}$". Cho ta $p \equiv 0; 1\pmod{7}$, điều này vô lí. Do vậy mọi ước nguyên tố không đồng dư $0, 1$ modulo $7$ của $a^{2} + b$ là ước nguyên tố của $a^{2} - 1$
Từ đây ta đi đến kết luận $(a^{2} + b) \mid 7(a^{2} - 1)$. Đặt $7(a^{2} - 1) = L(a^{2} + b) \iff (7 - L)(a^{2} + b) = 7(b + 1)$. Dễ thấy $L \ge 6$
$a^{2} + b = \frac{7(1 + b)}{7 - L} \implies \frac{7(1 + b)}{7 - L}\mid 7(b^{2} - 1) + 7(c + 1) \implies \frac{7(b + 1)}{7 - L}\mid 7(c + 1) \implies \frac{7(b + 1)}{7 - L}\mid 7(c^{2} - 1) + 7(1 + a) \implies \frac{7(b + 1)}{7 - L} \mid 7(a + 1)$. Từ đó có:
$$\frac{7(a + 1)}{a^{2} + b} \in \mathbb{Z}_{+}$$.
Thử từ $1$ đến $8$ có các nghiệm $(1; 1; 1); (6; 13; 370)$. Do $a, b, c$ đôi một nguyên tố cùng nhau nên chỉ có $(1; 1; 1)$ là bộ nghiệm duy nhất.




#616797 [Trường Xuân toán miền Nam]Vietnam TST 2016 Mock Test 1

Đã gửi bởi Ego on 25-02-2016 - 01:23 trong Thi HSG cấp Tỉnh, Thành phố. Olympic 30-4. Đề thi và kiểm tra đội tuyển các cấp.

Bài 3 ở 2 câu đầu không phức tạp quá.
Bài 3.
a) Chọn CSC $5, 17, 29, \cdots$.
Khi đó $\left(\frac{-1}{5}\right) = \left(\frac{-1}{17}\right) = \left(\frac{-1}{29}\right) = 1$. Nghĩa là tồn tại $a, b, c$ sao cho với $x \equiv a \pmod{5}$ thì $x^{2} + 1 \vdots 5$ (tương tự cho $17, 29$). Ta có hệ đồng dư:
$$\begin{cases} n\equiv a\pmod{5} \\ n\equiv b\pmod{17} \\ n\equiv c\pmod{29}\end{cases}$$
Do $(5; 17) = (17;29) = (29; 5) = 1$ nên theo định lý thặng dư Trung Hoa tồn tại $n$ để $n^{2} + 1 \vdots 5\times 17\times 29$
b) Gọi 3 số hạng luôn có mặt của dãy là $a, b, c$
Bổ đề 1. Mọi số nguyên dạng $4k + 3$ luôn có ước nguyên tố dạng $4n + 3$.
Bổ đề 2. Khồng tồn tại $p\in \mathbb{P}$ dạng $4k + 3$ sao cho $p\mid n^{2} + 1$ với $n$ tự nhiên nào đó.

TH1. $d = 10$. Giả sử tồn tại thoả có bộ chuẩn với $d = 10$. Nếu $a \equiv 0\pmod{2}$ thì $b, c$ cũng vậy. Do đó $4\mid abc$. Mặt khác, $n^{2} + 1 \equiv 1 \text{hoặc} 2\pmod{4}$. Do đó $abc\nmid n^{2} + 1$.
Nếu $a$ lẻ thì toàn bộ số hạng đều lẻ. Nếu $a \equiv 1\pmod{4}$ thì $b\equiv 3\pmod{4}$. Theo bổ đề 1 tồn tại số nguyên tố dạng $4k + 3$. Do đó $(4k + 2)\mid n^{2} + 1$, vô lí. Nếu $a\equiv 3\pmod{4}$ thì tương tự cũng có điều vô lí.
TH2. $d = 11$. Cũng giả sử ngược lại. Nếu $a \equiv 0\pmod{4}$ thì $4\mid abc$ và ta cũng có điều vô lí. Nếu $a \equiv 2\pmod{4}$ thì $c\equiv 0\pmod{4}$, dẫn đến $4\mid 8\mid abc$, vô lí. Nếu $a \equiv 3\pmod{4}$ thì cũng vô lí. Nếu $a\equiv 1\pmod{4}$ thì $c \equiv 3\pmod{4}$ cũng dẫn đến điều vô lí.
Tóm lại không có bộ chuẩn công sai $10$ hay $11$ (nói chung là không có bộ chuẩn công sai dạng $4k + 2$ hay $4k + 3$).
Đây là lời giải mình gõ đàng hoàng bên mathscope.
P.s: hôm nay mình thi ở trường, vô phòng thi cũng chán làm ra nốt câu c), cũng giống như Zaraki làm :-)



#616821 [Trường Xuân toán miền Nam]Vietnam TST 2016 Mock Test 1

Đã gửi bởi Ego on 25-02-2016 - 12:19 trong Thi HSG cấp Tỉnh, Thành phố. Olympic 30-4. Đề thi và kiểm tra đội tuyển các cấp.

Sẵn tiện rảnh mình sẽ post lời giải của mình. Có hai hướng đi và mình sẽ cung cấp một bổ đề mình nghĩ là mạnh hơn. Thật ra bài của bạn Zaraki việc chọn $k = 7$ là đẹp rồi.
Hướng 1. (mình vô tình vừa nghĩ được).
Với $7$ số thì tương tự như bạn Zaraki. Với $6$ số. Ta chọn 6 số cấp số cộng sao cho ước nguyên tố của chúng đều có dạng $4k + 1$ (rất nhiều cách chọn, có thể chọn ra cụ thể), gọi tích của chúng là $P$. Sử dụng đẳng thức $(a^{2} + b^{2})(c^{2} + d^{2}) = (ac + bd)^{2} + (ad - bc)^{2}$ ta có được tích của chúng dạng $u^{2} + v^{2}$. Chọn $u, v$ sao cho $\text{gcd}(u, v) = 1$. Khi đó xét $n' = (ur)^{2} + (vr)^{2} \vdots P$. Chọn $r$ sao cho $vr \equiv 1 \pmod{P}$. Do $\text{gcd}(u, v) = 1$ nên $\text{gcd}(v, P) = 1$. Do đó ta chọn được $r$. Khi đó chọn $n^{2} + 1$ sao cho $n \equiv ur \pmod{P}$. Từ đó có $n^{2} + 1 \equiv (ur)^{2} + (vr)^{2} \equiv 0 \pmod{P}$.
Hướng 2.
Bổ đề. Cho $p$ là số nguyên tố lẻ sao cho $\left(\frac{-1}{p}\right) = 1$. Khi đó luôn tồn tại $n$ để $p^{t} \mid n^{2} + 1$.
Chứng minh. Dĩ nhiên là tồn tại $n^{2} + 1 \vdots p$. Giả sử có tồn tại $v_{p}(n^{2} + 1) = t$, giả sử $n^{2} + 1 = p^{t}.r$ với $\text{gcd}(r, p) = 1$. Đặt $m = n + p^{t}.K$. Khi đó $m^{2} + 1 = n^{2} + 1 + 2np^{t}.K + p^{2t}.K^{2} \equiv r.p^{t} + 2nK.p^{t} = p^{t}(r + 2nK) \pmod{p^{t + 1}}$. Chọn $K$ sao cho $r + 2nK \vdots p$. Vậy bổ đề đúng theo quy nạp.

Dùng bổ đề ta có được điều phải chứng minh :-).
p.s: Hôm qua định dùng bổ đề này chứng minh cho 2 câu kia mà thấy hơi quá tay =))